Biology
Biology, 29.09.2019 06:30, heydisr

Quesliui 2
lp
which of the following statements is true?
lonotropic channels contain a neurotransmitter binding site but lack an ion channel as part
of its structure
binding of a neurotransmitter to an ionotropic receptor always results in an excitatoty
postsynaptic potential (epsp)
metabotropic receptors open an ion channel through the action of a g protein
metabotropic receptors are a type of ligand-gated channel
lonotropic receptors open an ion channel through the action of a g protein
question 3
1 nts

answer
Answers: 1

Other questions on the subject: Biology

image
Biology, 22.06.2019 00:50, ellaemtagedeane
How does deforestation affect the water cycle? a. it increases the amount of runoff b. it increases the amount of percolation c. it increases the amount of transpiration d. it increases the amount of precipitationa it increases the amount of runoff
Answers: 1
image
Biology, 22.06.2019 08:20, johnny08
Fungi are classified into how many groups? a. 1 group b. 2 groups c. 3 groups d. 4 groups
Answers: 1
image
Biology, 22.06.2019 11:00, suewignall
Astudent poured a solution of bromothymol blue indicator into three test tubes. then he placed an aquatic plant in two of the test tubes, as shown below. he placed a stopper on each test tube and placed them all in the dark for 24 hours. bromothymol blue turns from blue to yellow in the presence of co2
Answers: 2
image
Biology, 22.06.2019 20:50, sillyvanna
Assume that you are interested in separating short (200-400 bp) dna molecules from a pool of longer molecules in the 10,000-20,000 nucleotide range. you have two recipes for making your polyacrylamide gels: one recipe uses 1.5 percent agarose and would be considered a “hard gel,” while the other uses 0.5 percent agarose and would be considered a loose gel. which gel should you use to separate the short (200-400 bp) dna molecules from a pool of longer molecules in the 10,000-20,000 nucleotide range?
Answers: 3
Do you know the correct answer?
Quesliui 2
lp
which of the following statements is true?
lonotropic channels cont...

Questions in other subjects: